LSAT and Law School Admissions Forum

Get expert LSAT preparation and law school admissions advice from PowerScore Test Preparation.

 Administrator
PowerScore Staff
  • PowerScore Staff
  • Posts: 8917
  • Joined: Feb 02, 2011
|
#27911
Complete Question Explanation

Flaw in the Reasoning. The correct answer choice is (E)

In this stimulus, a columnist discusses recent research suggesting that vigorous exercise can significantly lower one’s chances for certain cardio-respiratory diseases. The columnist then concludes that one should ignore older studies that claim the same effect could be achieved by non-strenuous walking.

The question stem asks for the vulnerability in columnist’s reasoning. The columnist draws a conclusion based on the mistaken notion that because benefits can be derived in one manner, the same benefits can’t be achieved by other means:
  • Premise: ..... Strenuous walking reduces one changes of getting certain diseases.

    Flawed Conclusion: ..... Non-strenuous walking must not be effective.
Answer choice (A): The fact that other means might be effective in reducing certain illnesses does not weaken the columnist’s conclusion, because the columnist is only discussing exercise and whether or not the exercise needs to be vigorous. Whether there are other means of reducing the risks of cardio-respiratory diseases is irrelevant.

Answer choice (B): If the discussion were about overall health, this could be seen as weakening the columnist’s conclusion, but this discussion is only about the effect of exercise on cardio-respiratory diseases and whether that exercise must be vigorous. The fact that vigorous exercise may bring some outside risk does not play into an argument about its effects on the certain cardiovascular respiratory illnesses referenced in the stimulus.

Answer choice (C): Overlooking another possible benefit of vigorous exercise is not a flaw in this case. The discussion in the stimulus only concerns the beneficial effects of exercise on cardio-respiratory diseases and whether such exercise must be vigorous.

Answer choice (D): The discussion in the stimulus is not about perceptions of health, and we could only speculate as to the relationship between perception and physical manifestation. In any case, people who perceive themselves as healthy can still acquire cardio-respiratory diseases, and the columnist’s conclusion involves the effect of exercise. Since this answer choice does not discuss exercise at all, it does not reflect the flaw in the author’s reasoning.

Answer choice (E): This is the correct answer choice. The argument is weak because it fails to show that the conclusion of the recent report is better justified than an opposing conclusion reached in older studies. This mirrors our prephrased answer; no logical justification is offered for ignoring the old studies in favor of the new report.
 Jiya
  • Posts: 15
  • Joined: Aug 15, 2014
|
#16295
Hi,

In Q10 on Section 1 (LR) of Prep Test 51/ Dec 2006 - can you please explain how E is the correct answer? Doesn't the "only if" in the new report;s conclusion indicate that nothing but vigorous exercise yields the same benefit, thereby weakening the old report's conclusion?

Thanks!
 Ron Gore
PowerScore Staff
  • PowerScore Staff
  • Posts: 220
  • Joined: May 15, 2013
|
#16296
Hi Jiya!

You ask:
Jiya wrote:can you please explain how E is the correct answer? Doesn't the "only if" in the new report's conclusion indicate that nothing but vigorous exercise yields the same benefit, thereby weakening the old report's conclusion?
That's a great question, Jiya. After significant study, you no doubt have trained yourself not to question the premises of an argument, but rather how the conclusion is derived from the premises. Making the distinction between questioning the premises and questioning the conclusion derived from the premises can often be a tough distinction to make.

In this case, you are correct that the columnist tells us that the recent research report concludes that only vigorous exercise can lower one's chance of developing certain cardio-respiratory illnesses. And we won't doubt that the recent research report reached that conclusion. However, just because a recent report came to a different conclusion than an earlier report does not mean that the recent report is correct! For all we know, the recent report was faulty in some why, while the older studies are better supported and, therefore, more trustworthy.

The flaw in this argument is that the author believes the more recent report over the older reports without providing any reason to think that more recent report is more accurate. Answer choice (E) describes that flaw.

Please let me know if I can help further.

Thanks,

Ron
 Jiya
  • Posts: 15
  • Joined: Aug 15, 2014
|
#16330
I got it - thanks!
 Sherry001
  • Posts: 81
  • Joined: Aug 18, 2014
|
#20108
Hello;
Could someone kindly walk me through this problem? I had trouble choosing the correct answer. B seemed very attractive to me :(

P1: exercising vigorously can lower ones chances of developing certain cario-res illness.
P2: lower ones chances of cario-res illness -> exercising vigorously

C: One should not heed older studies purporting to show that nonstrenuous walking yields the same benefits.

My thinking before looking at the answer choices was something like.. " okay so we know the benefits of virgorous exercise . but are there any negative side effects?" and.. " we have no information about the older study, other than what were told in the conclusion . so how can we conclude , one should not heed older studies?"

A: seemed attractive because the stimulus said one can only achieve this benefit through vigorious exercise. but I eliminated this because NOT ALL cardi-resp required this, only certain ones did.

B: also seemed attractive. because we don't know any negative effects this may have. since were only told the benefits in the stimulus.

C:Nope, the stimulus said some/certain cardio resp disease .

D:Irrelevant

E: I didn't choose this, because I thought well... the only requirement in achieving these health benefits was thorough vigorous exercise. and if the older studies were nonstrenous , then it didn't meet our requirement .
 Steve Stein
PowerScore Staff
  • PowerScore Staff
  • Posts: 1153
  • Joined: Apr 11, 2011
|
#20124
Hi Sherry,

That's a good question. The columnist refers to research suggesting that vigorous exercise can lower certain cardio-respiratory health risks. Since the report claims such benefits can be derived only with exercise that's vigorous, the author concludes that previous studies should be ignored (those studies that suggested walking would yield the same benefits). The argument can be broken down as follows:
  • Premise: A recent study suggests that certain cardio benefits can be derived from exercise, but only if the exercise is vigorous.
  • Conclusion: Older studies, which claim to show that the same benefits can be derived from walking, should no longer be heeded.


It is not clear that the new study is any more reliable than the previous studies, but based on the results of the new study, the columnist jumps to the conclusion that all previous studies that show different results should be ignored. This is what correct answer choice (E) provides; the columnist fails to show that the recent study is any more reliable than the older studies, or that its conclusions are better justified.

I can certainly understand why answer choice (B) is appealing. If we were looking for some basis to simply attack the notion that vigorous exercise is better overall than non-vigorous exercise, this choice would fit the bill. But in this example, the question stem requires the answer choice that highlights the vulnerability in the columnist's argument. Recall the basic argument:
  • Since the recent report suggests that exercise has must be vigorous in order to derive cardio benefits, we should now ignore older reports claiming that walking can provide the same benefits.


Again, correct answer choice (E) specifies an inherent vulnerability in this argument, which is that the columnist concludes older reports should now be ignored, without providing any reason to believe that the new report is any more reliable or better justified than the older ones.

I hope that's helpful! Please let me know whether this is clear--thanks!

~Steve
 lathlee
  • Posts: 652
  • Joined: Apr 01, 2016
|
#46668
Is this General lack of offering relevant evidence Flaw?
 Adam Tyson
PowerScore Staff
  • PowerScore Staff
  • Posts: 5153
  • Joined: Apr 14, 2011
|
#47247
I'm no fan of trying to label every flaw, as if they will all fit neatly into one category or another, lathlee, and spending time trying to apply labels is, in my opinion, mostly a waste of time. The labels are there to help you, not constrain you. Sometimes you will see very clearly what category the flaw is, and that will help you to pick the best description for it (a Flaw question) or find the best match in the answers (Parallel Flaw). Other times, the label will be less obvious, and when that happens, instead of trying to attach a label, focus on why the argument is bad, why the premises don't support the conclusion. Describe it in your own words, instead of trying to make it fit into one of our boxes.

All that aside, I think it's fair to call this one a general lack of relevant evidence, since the author gave no reason - none! - to presume that the more recent report is better than the old one. Good job spotting that here!

Get the most out of your LSAT Prep Plus subscription.

Analyze and track your performance with our Testing and Analytics Package.